Oct 21

设$f_{n}:\mathbf{R}^{n}\to \mathbf{R}$为连续函数,且
\[ \lim_{n\to\infty}f_{n}(x)=f(x)\qquad \forall x\in \mathbf{R}^{n} \]
则$f$的连续点集在$\mathbf{R}^{n}$中为稠密的$G_{\delta}$集.

————————————————————————————————————————

证明:

我们首先注意到,若$G\subseteq \mathbf{R}$为开集,则$f^{-1}(G)$为$F_{\sigma}$集
事实上,由于$R$中的开集$G$是可数个构成区间的并,故不妨设$G$是一个开区间$(a,b)$.而我们知道
\[ \{x|f_{k}(x)\geq a+\varepsilon\}  \]
是闭集,由于$f_{k}(x)$的连续性,从而
\[ \{x|f(x)>a \}=\bigcup_{n=1}^{\infty}\bigcup_{m=1}^{\infty}\bigcap_{k=m}^{\infty}\left\{x|f_{k}(x)\geq a+\frac{1}{n}\right\} \]
是$F_{\sigma}$集.同理$\{x|f(x)<b\}$也是$F_{\sigma}$集合.而他们的交集
\[ f^{-1}((a,b))=\{x|f(x)>a \}\bigcap\{x|f(x)<b\} \]
也是$F_{\sigma}$集.为了证明$f(x)$的连续点是稠密的$G_{\delta}$集,我们只要证$f(x)$的不连续点是没有内点的$F_{\sigma}$集.记$D(f)$为$f(x)$的不连续点集合,任意的$a\in D(f)$,存在$p,q\in \mathbf{Q}$使得
\[ p<f(a)<q\]
\[ \Leftrightarrow\qquad a\in f^{-1}((p,q)) \]
\[ a\notin f^{-1}(R-(p,q)) \]
而存在点列$\{a_{n}\}$使得
\[ \lim_{n\to\infty}a_{n}=a,\qquad f(a_{n})\notin(p,q) \]
\[ \Leftrightarrow\qquad a_{n}\notin f^{-1}(p,q) \]
\[ \Leftrightarrow\qquad a_{n}\in f^{-1}(R-(p,q)), \lim_{n\to\infty}a_{n}=a \]
于是
\[ a\in \overline{f^{-1}(R-(p,q))}-f^{-1}(R-(p,q)) \]
所以
\[ D(f)=\bigcup_{p<q,\text{$p,q$为有理数}}[\overline{f^{-1}(R-(p,q))}-f^{-1}(R-(p,q))] \]
而$f^{-1}(R-(p,q))$为$G_{\delta}$集,故
\[ \overline{f^{-1}(R-(p,q))}-f^{-1}(R-(p,q)) \]
为$F_{\sigma}$集.并且$\overline{f^{-1}(R-(p,q))}-f^{-1}(R-(p,q))=\partial f^{-1}(R-(p,q))$是没内点的.它是可数个无内点的闭集的并,所以$D(f)$也是可数个无内点的闭集的并。由Baire定理,$D(f)$是无内点的$F_{\sigma}$集.所以$R^{n}\setminus D(f)$是稠密集合.

 

从这个结论可以看出,导函数的连续点也是稠密的,只要注意到

\[ f'(x)=\lim_{n\to\infty}\frac{f\left(x+\frac{1}{n}\right)-f(x)}{\frac{1}{n}} \]

就好.
 

Jun 12

设$a_{0}$和$a_{1}$是实数,且满足$a_{n+1}=a_{n}+\frac{2}{n+1}a_{n-1}$,证明序列$\left\{\frac{a_{n}}{n^2}\right\}$收敛,并求极限。


解:设
\[ S(x)=a_{0}+a_{1}x+\cdots+a_{n}x^{n}+\cdots \]

\begin{align*}
S(x)&=a_{0}+a_{1}x+\sum_{n=2}^{\infty}a_{n}x^{n}\\
&=a_{0}+a_{1}x+\sum_{n=1}^{\infty}a_{n+1}x^{n+1}\\
&=a_{0}+a_{1}x+\sum_{n=1}^{\infty}\left(a_{n}x^{n+1}+\frac{2}{n+1}a_{n-1}x^{n+1}\right)\\
&=a_{0}+a_{1}x+x(S(x)-a_{0})+2\int_{0}^{x}tS(t)dt
\end{align*}
两边对$x$求导,得到微分方程
\[ (x-1)S'(x)+(2x+1)S(x)+a_{1}-a_{0}=0 \]
注意到初值$S_{0}=a_{0}$,解这个ODE,得到
\[ S(x)=\frac{1}{4}\cdot\left[\frac{(2x^2-6x+5)(a_{0}-a_{1})+(5a_{1}-9a_{0})e^{-2x}}{(x-1)^3}\right]\]
我们有展开式
\[ \frac{1}{(1-x)^{3}}=\sum_{k=0}^{\infty}\frac{(k+2)(k+1)}{2}x^{k} \]
\[ e^{-2x}=\sum_{k=0}^{\infty}\frac{(-2)^{k}}{k!}x^{k} \]

\[ (2x^2-6x+5)\cdot\frac{1}{(1-x)^3}=5+9x+\sum_{n=2}^{\infty}\frac{1}{2}(n+5)(n+2)x^{n} \]
\[ e^{-2x}\cdot\frac{1}{(1-x)^3}=\sum_{n=0}^{\infty}c_{n}x^{n} \]
其中
\[ c_{n}=\sum_{k=0}^{n}\frac{(-2)^{k}(n-k+2)(n-k+1)}{2\cdot k!} \]
于是
\[ S(x)=\frac{1}{4}(a_{1}-a_{0})\left(5+9x+\sum_{n=2}^{\infty}\frac{1}{2}(n+5)(n+2)x^{n}\right)+\left(\frac{9}{4}a_{0}-\frac{5}{4}a_{1} \right)\left(\sum_{n=0}^{\infty}c_{n}x^{n}\right)\]
对比$x^{n}$项的系数,得到
\[a_{n}=\frac{1}{8}(n+5)(n+2)(a_{1}-a_{0})+\left(\frac{9}{4}a_{0}-\frac{5}{4}a_{1}\right)\left(\sum_{k=0}^{n}\frac{(-2)^{k}(n-k+2)(n-k+1)}{2\cdot k!}\right)\]
下面来计算
\[ \lim_{n\to\infty}\frac{a_{n}}{n^2} \]
显然有
\[ \lim_{n\to\infty}\frac{\frac{1}{8}(n+5)(n+2)(a_{1}-a_{0})}{n^2}=\frac{1}{8}(a_{1}-a_{0}) \]
又注意到
\begin{align*}
&\frac{1}{n^2}\sum_{k=0}^{n}\frac{(-2)^{k}}{2\cdot k!}(n-k+2)(n-k+1)\\
&= \frac{1}{n^2}\sum_{k=0}^{n}\frac{(-2)^{k}}{2\cdot k!}[n^2+3n-2kn+(k-2)(k-1)]\\
&=\sum_{k=0}^{n}\frac{(-2)^{k}}{2\cdot k!}+o(1)\\
&\to \frac{e^{-2}}{2}\qquad (n\to\infty)
 \end{align*}
 所以
 \[ \lim_{n\to\infty}\frac{a_{n}}{n^2}=\left(\frac{9}{8}e^{-2}-\frac{1}{8}\right)a_{0}+\left(\frac{1}{8}-\frac{5}{8}e^{-2}\right)a_{1} \]

Jun 10

设$P_{0}(x)=\sqrt{x}$,$P_{n+1}(x)=\frac{1}{2}P_{n}^{2}(x)+(1-\sqrt{x})P_{n}(x)$,证明
\[ P_{n}(x)\rightrightarrows 0\qquad (x\in[0,1]) \]

证明:这个题源自周民强的习题集,实际是越南1989年的一个竞赛题的改编,但解答几乎都是采用了数学归纳法,外表看似合理,但是却难以想到。昨天看见西哥给出了不是归纳的办法,赞一个。现收录如下

不难发现$P_{n}(x)\geq P_{n+1}(x)$,和

\[ 0\leq P_{n}(x)\leq \sqrt{x} \]

现在注意到

\[ P_{n+1}(x)=P_{n}(x)\left(\frac{1}{2}P_{n}(x)+1-\sqrt{x}\right)\leq P_{n}(x)\left(1-\frac{1}{2}\sqrt{x}\right)\]

就有

\[ P_{n+1}(x)\leq P_{0}(x)\left(1-\frac{1}{2}\sqrt{x}\right)^{n}=\left(\frac{n}{2}\right)^{n}\sqrt{x}\left(\frac{2}{n}-\frac{1}{n}\sqrt{x}\right)^{n}\leq \left(\frac{n}{2}\right)^{n}\left(\frac{2}{n+1}\right)^{n+1}\leq \frac{2}{n+1} \]

显然就有

\[ P_{n}(x)\rightrightarrows 0\qquad (x\in[0,1]) \]

Jun 10

设$f(x)$在$[0,\pi]$上可微,且满足$(f'(x))^2 \in R[a,b]$,若$\displaystyle \int_{0}^{\pi}f(x)dx=0$,则
\[ \int_{0}^{\pi}f^{2}(x)dx\leq \int_{0}^{\pi}(f'(x))^2 dx \]


证明:我们把$f(x)$以$\pi$为周期先偶延拓到$[-\pi,\pi]$,再以$2\pi$为周期延拓到$\mathbb{R}$,这时$2k\pi,k\in \mathbf{N}^{+}$是$f(x)$的周期,注意到
\[ f(x+2\pi)=f(x),f(x)=f(-x) \]
两边对$x$求导,得到
\[ f'(x+2\pi)=f'(x),f'(x)=-f'(-x) \]
就是说$2k\pi,k\in \mathbf{N}^{+}$也是$f'(x)$的周期,而
\[ \int_{0}^{\pi}f^{2}(x)dx=\int_{-\pi}^{0}f^{2}(x)dx=\int_{\pi}^{2\pi}f^2(x)dx \]

\[ \int_{0}^{\pi}(f'(x))^2dx=\int_{-\pi}^{0}(f'(x))^2dx=\int_{\pi}^{2\pi}(f'(x))^2dx \]
不等式显然等价于
\[ \int_{0}^{2\pi}f^{2}(x)dx\leq \int_{0}^{2\pi}(f'(x))^2dx \]
设$f(0)=f(2\pi)=a$,这时,注意到
\[ 0\leq (f(x)-a)^2\cot{x}\leq \left(\int_{0}^{x}f'(t)dt\right)^{2}\frac{1}{\sin{x}}\leq \int_{0}^{x}dt\int_{0}^{x}(f'(t))^{2}dt\frac{1}{\sin{x}}=\frac{x}{\sin{x}}\int_{0}^{x}(f'(t))^{2}dt\to 0 (x\to 0^{+}) \]

\[ \lim_{x\to 0}(f(x)-a)^{2}\cot x=0 \]
同理也有
\[ \lim_{x\to 2\pi}(f(x)-a)^{2}\cot x=0 \]
不妨延拓定义
\[ (f(x)-a)^{2}\cot x\bigg|_{x=0}=(f(x)-a)^{2}\cot x\bigg|_{x=2\pi}=0 \]
现在,我们有
\begin{align*}
&\int_{0}^{2\pi}\bigg((f'(x))^2-(f(x)-a)^2-[f'(x)-(f(x)-a)\cot x]^{2}\bigg)dx\\
&=\int_{0}^{2\pi}\bigg(-(f(x)-a)^2\csc^{2}x+2f'(x)(f(x)-a)\cot x\bigg)dx\\
&=(f(x)-a)^{2}\cot x\bigg|_{0}^{2\pi}\\
&=0
\end{align*}

\[ \int_{0}^{2\pi}[(f'(x))^2-(f(x)-a)^2]dx=\int_{0}^{2\pi}[f'(x)-(f(x)-a)\cot x]^{2}dx\geq 0 \]
而注意到
\[ \int_{0}^{2\pi}f(x)dx=0 \]则
\[ \int_{0}^{2\pi}[(f'(x))^2-(f(x)-a)^2]dx=\int_{0}^{2\pi}((f'(x))^2-(f(x))^2)dx-2\pi a^2\geq 0 \]
就是说
\[ \int_{0}^{2\pi}(f'(x))^2dx\geq \int_{0}^{2\pi}(f(x))^2dx+2\pi a^2\geq \int_{0}^{2\pi}(f(x))^2dx \]
Hence we are done!

Jun 9

本文转载自xida博客朝花夕拾,原文网址:http://xidalapuda.diandian.com/post/2014-03-02/40061132729

 

Jordan 标准形定理是线性代数中的基本定理,专门为它写一篇长文好像有点多余:这方面的教材讲义实在是太多了!一个陈旧的定理还能写出什么新意来呢?

理由有两个。第一个原因是我曾经在给学生讲这个定理的时候,突然发现不知道该怎么启发学生为好。虽然我知道 Jordan 标准形定理的很多种证法,照念几个不在话下,但是感觉有点疙疙瘩瘩的:怎么才能说清定理背后的想法,让学生觉得定理的成立是顺理成章的呢?于是我知道我对这个定理的理解还有模糊的地方。

第二个原因是 Jordan 块有一个重要的代数性质是通常教材中不讲的,而这个性质是代数学中一类重要而常见的性质的雏形,这就是不可分解性。与之对应的是可对角化的线性变换的完全可约性。从一开始就让学生接触这些现象是有好处的。



~~~~~~~~~~~~~~~~~~~~~~~~~~~~~~~~~~~~~~~~~~~~~~~~~~~~~~~~~



我们从中学就知道整数环和多项式环有唯一因子分解定理:每个整数可以唯一地分解为素数的乘积,每个(域上的)多项式可以唯一地分解为不可约多项式的乘积。在数学里面有很多这样的唯一分解定理,而我们现在想知道:有没有所谓的 "线性变换的唯一分解定理" 呢?可以猜测如果有这样的定理存在,那么大概可以表述为如下的样子:



线性变换的唯一分解定理(粗糙的版本):设 $V$ 是域 $F$ 上的有限维向量空间,$A$ 是 $V$ 上的线性变换,则 $A$ 可以唯一地分解为若干个 "简单的" 线性变换的组合,而且这些 “简单的” 线性变换本身不能再分解。



这个表述很不清楚,整数和多项式的分解就是表示为因子的乘积,那么什么是线性变换的分解呢?什么又是不可分解的线性变换呢?正确的概念是直和:



设 $T$ 是 向量空间 $V$ 上的线性变换,如果 $V$ 可以分解为一些非平凡的子空间的直和 $V=V_1\oplus\cdots\oplus V_k$,使得每一个 $V_i$ 都是 $T-$ 不变的子空间,则称 $T$ 是可以分解的; 如果 $V$ 不存在这样的分解,则称 $T$ 是不可分解的线性变换。



这样我们就可以比较准确的表述线性变换的唯一分解定理了:



线性变换的唯一分解定理(修正的版本):设 $V$ 是域 $F$ 上的有限维向量空间,$T$ 是 $V$ 上的线性变换,则 $T$ 可以唯一地分解为若干个不可分解的线性变换的直和。



这里有一个很严重的问题需要说明:在一般的域 $F$ 上 研究 "不可分解" 的线性变换是一个棘手的多的问题,这个问题的解决要用到我们后面要学的有理标准形,而在复数域上问题就简单很多,这就是 Jordan 标准形做的事情。所以在本文中,域 $F$ 都假定为复数域 $\mathbb{C}$。



那么什么样的线性变换算是不可分解的线性变换呢?



最简单也是最重要的例子就是移位算子:假设 $T$ 在 $V$ 的一组基 $\{v_1,\cdots,v_n\}$ 的作用是一个向右的移位:

\[ T:\quad v_n\rightarrow v_{n-1}\rightarrow\cdots\rightarrow v_1\rightarrow0.\]

则称 $T$ 是一个移位算子。$T$ 在这组基下的矩阵

\[J_0=\begin{pmatrix}0&1&&\\&\ddots&\ddots&\\&&0&1\\&&&0\end{pmatrix}.\]

$J_0$ 叫做特征值为 0 的 Jordan 块。注意 $T$ 是一个幂零算子:$T^n=0$,它仅有唯一的特征值 0。



当然需要说明移位算子 $T$ 确实是不可分解的线性变换。如果 $V=W\oplus N$ 为两个非平凡 $T-$ 不变子空间的直和,则 $T$ 在 $W$ 和 $N$ 上各有一个特征值为 0 的特征向量,因此齐次线性方程组 $TX=0$ 的解空间至少包含两个线性无关的向量。但是 $T$ 的秩是 $n-1$,因此 $TX=0$ 的解空间是 1 维的,这就导致了矛盾。



用同样的方法可以说明给移位算子 $T$ 加上一个数乘变换以后得到的仍然是不可分解的线性变换:设 $\lambda\in\mathbb{C}$,$S=T+\lambda I$,则 $S$ 也是不可分解线性变换,其对应的矩阵

\[J_\lambda=\begin{pmatrix}\lambda&1&&\\&\ddots&\ddots&\\&&\lambda&1\\&&&\lambda\end{pmatrix}\]

叫做特征值为 $\lambda$ 的 Jordan 块。



现在我们已经找到了一族不可分解的线性变换,那么它们是否就构成了全部的线性变换呢?答案是肯定的,这就是 Jordan 标准形定理:



Jordan 标准形定理:设 $T$ 是 $\mathbb{C}$ 上有限维向量空间 $V$ 上的线性变换,则存在 $V$ 的一组基使得 $T$ 在这组基下的矩阵为 Jordan 块的直和:\[T=J_{\lambda_1}\oplus\cdots\oplus J_{\lambda_r}.\]这种分解是唯一的,意思是如果存在 $V$ 的另一组基使得 $T$ 的矩阵也是 Jordan 块的直和 \[T=J_{\mu_1}\oplus\cdots\oplus J_{\mu_s},\]则 $r=s$ 且适当重排后有 $J_{\lambda_i}=J_{\mu_i}$。



定理的结论包含存在性和唯一性两部分,我们先来处理存在性的证明。



~~~~~~~~~~~~~~~~~~~~~~~~~~~~~~~~~~~~~~~~~~~~~~~~~~~~~~~~~~



第一步:转化为幂零的情形



定理 【广义特征子空间分解】:设 $T$ 的特征多项式为 $f(x)$,而且 $f(x)$ 在复数域上分解为一次因式的乘积\[f(x)=(x-\lambda_1)^{n_1}\cdots(x-\lambda_k)^{n_k},\]

这里的 $\lambda_i$ 互不相同。令 $V_i=\ker (T-\lambda_i I)^{n_i}$,则每个 $V_i$ 都是 $T-$ 不变子空间而且\[ V=V_1\oplus\cdots\oplus V_m.\]

这样就把 $V$ 分解为一些不变子空间 $V_i$ 的直和,$T$ 限制在每个 $V_i$ 上只有单一的特征值 $\lambda_i$。



证明:显然 $V_i$ 都是 $T-$ 不变子空间。要证明 $V$ 是它们的直和,我们先从一个特别的结论开始:



对每个 $1\leq i\leq k$ 都存在多项式 $\pi_i(x)$ 使得 $\pi_i(x)\equiv1\mod (x-\lambda_i)^{n_i}$,但是对其它 $j\ne i$ 有 $\pi_i(x)\equiv0\mod (x-\lambda_j)^{n_j}$。线性变换 $\pi_i(T)$ 不是别的,正是 $V$ 到子空间 $V_i$ 的投影。



由于所有 $(x-\lambda_i)^{n_i}$ 的根互不相同,因而两两互素,所以根据中国剩余定理满足要求的 $\pi_i(x)$ 是存在的。显然 $\pi_i(T)$ 在 $V_i$ 上是恒等变换,而在其余的 $V_j\ne V_i$ 上是 0。$\pi(x)=\pi_1(x)+\cdots+\pi_k(x)$ 模任何 $(x-\lambda_i)^{n_i}$ 都是 1,因此 $\pi(x)-1$ 可以被 $T$ 的特征多项式 $f(x)$ 整除,从而 $\pi(T)-I$ 在 $V$ 上是零变换,这就证明了 $\pi(T)$ 是 $V$ 上的恒等变换。对任何 $v\in V$,\[v=\pi(T)v=\pi_1(T)v+\cdots+\pi_k(T)v.\]

我们来说明 $\pi_i(T)v\in V_i$,从而 $V=V_1+\cdots+V_k$。这是因为 $(x-\lambda_i)^{n_i}\pi_i(x)$ 可以被 $f(x)$ 整除,因此 $(T-\lambda_i)^{n_i}\pi_i(T)v=0$,这就证明了 $\pi_i(T)v\in V_i$。



我们再来说明这是直和。如果 $v_i\in V_i$ 满足 $v_1+\cdots+v_k=0$,用 $\pi_i(T)$ 作用在左边得到(根据前面的分析,$\pi_i(T)$ 在 $V_i$ 上是恒等变换而在其它 $V_j$ 上是 0)

\[\pi_i(T)v_1+\cdots+\pi_i(T)v_k=\pi_i(T)v_i=v_i=0,\]

由 $i$ 的任意性得到 $v_1=\cdots=v_k=0$,这就证明了是直和。



用中国剩余定理来构造特殊的算子(通常是给定的算子 $T$ 的多项式)是一个普遍而重要的技巧,这里的证明也许有点高端但却是最简洁的。

现在我们只需要考虑单个子空间 $V_i$。令 $N=T-\lambda_i$,则 $N$ 在 $V_i$ 上是幂零线性变换:$N^{n_i}=0$,这样问题归结为分析幂零线性变换 $N$ 的结构。

幂零线性变换更简单的原因是它可以表示为移位算子的直和,而移位算子的结构非常简单。

~~~~~~~~~~~~~~~~~~~~~~~~~~~~~~~~~~~~~~~~~~~~~~~~~~~~~~~~~~



第二步:对幂零变换的情形加以证明



设 $N$ 是 $V$ 上的幂零线性变换,要证明存在 $V$ 的一组基,使得 $N$ 的矩阵是若干 Jordan 块的和。注意一个 Jordan 块对应的是一个移位轨道\[ v\rightarrow Nv\rightarrow \cdots \rightarrow N^kv\rightarrow 0.\]

我们要证明存在若干条这样的互不相交的轨道,这些轨道所包含的全部非零向量构成 $V$ 的一组基。



这一步的证明方法很多,但是相差不是很大,具体喜欢那种要看个人主观,这里介绍的是最简单也是最容易被初学者接受的一种。



对 $V$ 的维数 $\dim V$ 归纳,$\dim V=1$ 时显然结论成立。

现假设结论对所有维数小于 $\dim V$ 的向量空间成立,我们考虑 $V$ 的像空间 $N(V)$。这是一个 $N-$ 不变子空间,且由于 $N$ 是幂零线性变换所以 $\dim N(V)<\dim V$,所以可以对子空间 $N(V)$ 使用归纳假设:存在 $N(V)$ 的一组基如下,它们构成 $q$ 条不相交的轨道 $\mathcal{O}_1,\cdots,\mathcal{O}_q$:

\[\begin{array}{l}&v_{1,1}\rightarrow v_{1,2}\rightarrow\cdots\rightarrow v_{1,n_1}\rightarrow 0.\\&v_{2,1}\rightarrow v_{2,2}\rightarrow\cdots\rightarrow v_{2,n_2}\rightarrow 0.\\&\cdots\cdots\cdots\\& v_{q,1}\rightarrow v_{q,2}\rightarrow\cdots\rightarrow v_{q,n_q}\rightarrow 0.\end{array}\]

由于 $v_{i,1}\in N(V)$ 因此可以设 $v_{i,1}=Nw_i$,从而我们得到一组更长的轨道(就是在前面加上一项)

\[\begin{array}{l}&w_1\rightarrow v_{1,1}\rightarrow v_{1,2}\rightarrow\cdots\rightarrow v_{1,n_1}\rightarrow 0.\\&w_2\rightarrow v_{2,1}\rightarrow v_{2,2}\rightarrow\cdots\rightarrow v_{2,n_2}\rightarrow 0.\\&\cdots\cdots\cdots\\&w_q\rightarrow v_{q,1}\rightarrow v_{q,2}\rightarrow\cdots\rightarrow v_{q,n_q}\rightarrow 0.\end{array}\]

那么这些新轨道包含的向量是否构成 $V$ 的一组基?答案是我们还要补上一些在 $V$ 中长度是 1,但是在 $N(V)$ 中 "消失" 了的轨道:注意 $\{v_{1,n_1},\cdots,v_{q,n_q}\}$ 是 $\ker N$ 中的线性无关元,但是 $\ker N$ 还可能有其它的基向量。将它们扩充为 $\ker N$ 的一组基

\[ \{ v_{1,n_1},\cdots,v_{q,n_1}\}\cup \{ w_{q+1},\cdots,w_{K}\}\quad K=\dim\ker N.\]

从而我们最终得到下面的轨道图:

\[\begin{array}{r}\mathbf{w_1\rightarrow} v_{1,1}\rightarrow v_{1,2}\rightarrow\cdots\rightarrow v_{1,n_1}\rightarrow 0.&\\\mathbf{w_2\rightarrow} v_{2,1}\rightarrow v_{2,2}\rightarrow\cdots\rightarrow v_{2,n_2}\rightarrow 0.&\\\cdots\cdots\cdots&\\\mathbf{w_q\rightarrow} v_{q,1}\rightarrow v_{q,2}\rightarrow\cdots\rightarrow v_{q,n_q}\rightarrow 0.&\\\mathbf{w_{q+1}\rightarrow} 0.&\\\cdots\cdots&\\\mathbf{w_K\rightarrow0}.\end{array}\]

你可以看到 $w_{q+1},\ldots,w_K$ 正是那些在 $V$ 中长度为 1,但是在 $N(V)$ 中消失了的轨道。

最后只剩下验证这些向量确实构成 $V$ 的一组基。显然这些向量一共有 $\dim N(V)+\dim\ker N=\dim V$ 个,所以只要说明它们是线性无关的。

假设有线性关系

\[\cdots+(c_0w_i+c_1v_{i,1}+\cdots+c_{n_i}v_{i,n_i})+\cdots+\sum_{j=q+1}^K d_jw_j=0,\]

我们要说明出现在上式中的所有系数都是 0。左边用 $N$ 作用得到

\[\cdots+(c_0v_{i,1}+c_1v_{i,2}+\cdots+c_{n_i-1}v_{i,n_i})+\cdots=0.\]

这是一个关于 $N(V)$ 的一组基的一个线性关系,于是 $c_0=\cdots=c_{n_i-1}=0$,从而剩下的线性关系为

\[\cdots+c_{n_i}v_{i,n_i}+\cdots+\sum_{j=q+1}^K d_jw_j=0.\]

而这是一个关于 $\ker N$ 的一组基的一个线性关系,于是 $c_{n_i}=d_{q+1}=\cdots=d_K=0$,从而所有的系数都是 0,这就完成了 Jordan 标准形存在性的证明。



~~~~~~~~~~~~~~~~~~~~~~~~~~~~~~~~~~~~~~~~~~~~~~~~~~~~~~~~~~



分解唯一性的证明:



最后我们还剩下分解唯一性定理的证明,这部分要简单许多,主要是利用了 Jordan 块的一个很特殊的性质:设

\[J_0=\begin{pmatrix}0&1&&\\&\ddots&\ddots&\\&&0&1\\&&&0\end{pmatrix}_{n\times n}\]

是一个 0 特征值的 Jordan 块,则 $J_0^2$ 就是把斜对角线上的 1 向右上方平移一步,$J_0^3$ 就是向右上方平移两步,以此类推,$J_0^{n-1}$ 变成

\[\begin{pmatrix}0&\cdots&1\\&\ddots&\vdots\\&&0\end{pmatrix},\]

最终 $J_0^n=0$。用这个规则我们可以计算出对任何 $\lambda\in\mathbb{C}$ 和 $m\in\mathbb{Z}^+$,$T$ 的 Jordan 标准形中 $m$ 阶 Jordan 块 $J_{\lambda,m}$ 的个数 $n_m$ 来:\[ n_m=\text{rank}(T-\lambda I)^{m-1}-2\cdot\text{rank}(T-\lambda I)^{m}+\text{rank}(T-\lambda I)^{m+1}.\]

道理是这样的:以 $\lambda=0$ 为例子来计算。会算 0 特征值 Jordan 块的个数,你就会算任何特征值的 Jordan 块的个数。设 $T$ 的一个 Jordan 标准形为

\[ T= \left(\bigoplus_{k\geq1}n_k J_{0,k}\right) \bigoplus_{\mu\ne0}J_\mu,\]

那么 $T^m$ 就是

\[T^{m}= \left(\bigoplus_{k\geq1}n_k J_{0,k}^{m}\right) \bigoplus_{\mu\ne0}J_{\mu}^{m}.\]

注意后半部分 $\oplus_{\mu\ne0}J^m_\mu$ 对任何 $m$ 都是保持满秩的,因此这部分的秩始终不变。前面的部分中所有阶数小于等于 $m$ 的 Jordan 块 $J_{0,k}(k\leq m)$ 的 $m$ 次幂都变成了 0 矩阵,$J_{0,m+1}^m$ 的秩是 1; $J_{0,m+2}^m$ 的秩是 2 . . . 依次类推,所以

\[ \text{rank}T^m=n_{m+1}\cdot1+n_{m+2}\cdot2+\cdots +\text{rank}\bigoplus_{\mu\ne0}J_\mu^m.\]

同理

\[ \text{rank}T^{m+1}=n_{m+2}\cdot1+n_{m+3}\cdot2+\cdots +\text{rank}\bigoplus_{\mu\ne0}J_\mu^{m+1}.\]

因此

\[\text{rank}T^m-\text{rank}T^{m+1}=n_{m+1}+n_{m+2}+\cdots,\]

仍然同理

\[\text{rank}T^{m-1}-\text{rank}T^{m}=n_{m}+n_{m+1}+\cdots,\]

所以

\[n_m=\text{rank}T^{m-1}-2\cdot\text{rank}T^{m}+\text{rank}T^{m+1}.\]

现在你可以看到 $n_m$ 的表达式不依赖于具体的基的选择,仅依赖于线性变换自身的相似不变量,所以 $T$ 的 Jordan 标准形在只差一个排列的意义下是唯一的。



~~~~~~~~~~~~~~~~~~~~~~~~~~~~~~~~~~~~~~~~~~~~~~~~~~~~~~~~~~



一个很有意思的问题是,给定

\[J_\lambda=\begin{pmatrix}\lambda&1&&\\&\ddots&\ddots&\\&&\lambda&1\\&&&\lambda\end{pmatrix}_{n\times n}\]

为一个特征值 $\lambda$ 的 Jordan 块,计算其 $k$ 次幂 $J_\lambda^k$ 的 Jordan 标准形。



当 $\lambda\ne0$ 时,

\[J_\lambda^k = \begin{pmatrix}\lambda^k &k\lambda^{k-1}&\ddots &\\&\lambda^k&\ddots&\ddots\\&&\ddots&k\lambda^{k-1}\\&&&\lambda^k\end{pmatrix}.\]

(你知道怎样计算 $J_\lambda^k$ 吗?记住这个技巧:把多项式 $x^k$ 在 $\lambda$ 处 Taloy 展开:\[x^k=(x-\lambda)^k+a_{k-1}(x-\lambda)^{k-1}+a_1(x-\lambda)+a_0,\]然后将 $J_\lambda$ 代入即可。)



和 Jordan 块不可分解性的证明完全一样,我们发现 $J_\lambda^k-\lambda^k I$ 的秩是 $n-1$,因此方程组 $J_\lambda^kX=\lambda^k X$ 的解空间是 1 维的,从而 $J_\lambda^k$ 是不可分解的,因此其 Jordan 标准形只有一块,就是

\[\begin{pmatrix}\lambda^k&1&&\\&\ddots&\ddots&\\&&\lambda^k&1\\&&&\lambda^k\end{pmatrix}_{n\times n}.\]

最有意思的情形发生在 $\lambda=0$ 时。这个时候 Jordan 会均匀的碎裂为一些小的 Jordan 块的和。

这个时候 $J_0$ 是一个移位算子:

\[J_0:\quad v_n\rightarrow v_{n-1}\rightarrow \cdots \rightarrow v_1\rightarrow 0.\]

整个轨道只有一条。但是 $J_0^k$ 则是 $k$ 步 $k$ 步地跳:

\[J_0^k:\quad \left\{ \begin{array}{l} v_n\rightarrow v_{n-k}\rightarrow \cdots \rightarrow0,\\v_{n-1}\rightarrow v_{n-1-k}\rightarrow \cdots\rightarrow 0,\\\cdots\\v_{n-k+1}\rightarrow v_{n-2k+1}\rightarrow \cdots \rightarrow 0.\end{array}\right.\]

所以 $J_0^k$ 有 $k$ 条轨道,每个轨道都是一个 Jordan 块,即 $J_0^k$ 的标准形中有 $k$ 个 Jordan 块。设 $n=qk+r$,这里 $0\leq r< k$,则这 $k$ 个 Jordan 块中有 $r$ 个是 $q+1$ 阶的,$k-r$ 个是 $q$ 阶的。



举个例子就明白了,一个 8 阶的 0 特征值 Jordan 块 $J_0$,$J_0^3$ 的 Jordan 标准形是什么样子的?这个时候 $J_0^3$ 有 3 个轨道 $\{v_8,v_5,v_2\}$, $\{v_7,v_4,v_1 \}$, $\{v_6,v_3\}$,所以 $J_0^3$ 的 Jordan 标准形有 2 个 3 阶的 Jordan 块和 1 个 2 阶的 Jordan 块。

总结一下:零特征值的 Jordan 块的高次幂一定会分裂,而且是尽可能均匀的分裂;非零特征值的 Jordan 块的任意次幂都不会分裂。



一个不可约的代数结构,在某种限制或者扩张的意义下却能均匀的 "碎裂",这是代数学中一个常见而重要的现象。比如设 $f$ 是一个有理数域 $\mathbb{Q}$ 上的不可约多项式,$F$ 是 $Q$ 的一个正规扩域,则如果 $f$ 在 $F$ 上是可约的,那么 $f$ 必然分解成一些次数相同的多项式的乘积:\[f=f_1f_2\cdots f_r,\quad \deg f_1=\cdots=\deg f_r.\]

类似的还有代数数论中素理想的分解,群表示论中不可约表示(在诱导和限制下)的分解,代数几何中不可约代数簇的分解等等。